偏差値30からの早慶圧勝の個別指導塾 HIRO ACADEMIA

偏差値30からの早慶圧勝の個別指導塾 HIRO ACADEMIA

  • 資料請求
  • カウンセリング

2016年早稲田大学商学部|過去問徹底研究 大問1

2019.09.30

方針の立て方 (1) 典型問題であり,特筆事項なし. (2) 代数方程式の有理数解に着目していることから解法を得る. (3) 実際に考えてみることで解法を得る.ただし,を正2016角形で考えるのは難しいため,正4角形や,正5角形,正6角形などで考えてみる.そうすると,約数の問題であることに気付ける.

  • …続きを読む
  • 方針の立て方
    (1)
    典型問題であり,特筆事項なし.

    (2)
    代数方程式の有理数解に着目していることから解法を得る.

    (3)
    実際に考えてみることで解法を得る.ただし,Pを正2016角形で考えるのは難しいため,正4角形や,正5角形,正6角形などで考えてみる.そうすると,約数の問題であることに気付ける.

    (4)
    最初の式のままでは考えづらいため,変形を試みる.そこで,積和の公式を使って,三角関数の積の形を和の形に直す.
    本問に限らず,数学では,和から積への変形,積から和への変形をすることで解法が見えることが多いため,困ったときにはこのような変形をとりあえず試みることを心がけよう.

    解答例
    (1)ア:1024
    (2)イ:5
    (3)ウ:3528
    (4)エ:-1008

    解説
    (1)
    合同式の法は全部2016とする.
    2^{11}=2048\equiv32=2^5
    である.
    \therefore2^{100}=2\cdot\left(2^{11}\right)^9\equiv2\cdot\left(2^5\right)^9=2^2\cdot\left(2^{11}\right)^4≡22⋅254=2112≡252=1024……(答)

    (2)
    有理数解は全て

    の形で表せる.よって,1以上の有理数解の候補は,1,\frac{3}{2},3である.
    (ⅰ)x=1が解になると仮定して,方程式に代入すると
    2\cdot1^3-a\cdot1^2+b\cdot1+3=0\Leftrightarrow a=5+b\geqq5+1=6
    (ⅱ)x=\frac{3}{2}が解になると仮定して,方程式に代入すると
    2\cdot\left(\frac{3}{2}\right)^3-a\cdot\left(\frac{3}{2}\right)^2+b\cdot\frac{3}{2}+3=0\Leftrightarrow a=\frac{2b+13}{3}\geqq\frac{2\cdot1+13}{3}=5
    (ⅱ)x=3が解になると仮定して,方程式に代入すると
    2\cdot3^3-a\cdot3^2+b\cdot3+3=0\Leftrightarrow a=\frac{b+19}{3}\geqq\frac{1+19}{3}=\frac{20}{3}
    以上(ⅰ)~(ⅲ)より,求めるaの最小値は,5……(答)

    (3)
    2016の任意の約数をaとする.
    Pの頂点を結ぶことで作ることができる正多角形は,正a角形(a=1,2を除く)のみである.
    a角形の作り方は\frac{2016}{a}通りあるが,\frac{2016}{a}も2016の約数となる.
    よって,求める個数は,2016の約数の和から,a=1,2のときの分\frac{2016}{a}=2016,1008を除いた個数となる.2016=2^5\cdot3^2\cdot7より,2016の約数の和は,
    \sum_{z=0}^{1}\sum_{y=0}^{2}\sum_{x=0}^{5}{2^x\cdot3^y\cdot7^z}=\left(7^0+7^1\right)\left(3^0+3^1+3^2\right)\left(2^0+2^1+2^2+2^3+2^4+2^5\right)=8\cdot13\cdot63=6552
    よって,求める個数は,
    6552-2016-1008=3528個……(答)

    (4)
    \left(\sum_{k=1}^{2016}{k\sin{\frac{\left(2k-1\right)\pi}{2016}}}\right)\sin{\frac{\pi}{2016}}=\sum_{k=1}^{2016}\left\{k\sin{\frac{\left(2k-1\right)\pi}{2016}}\sin{\frac{\pi}{2016}}\right\}
    ここで,積和の公式より,
    \sin{\frac{\left(2k-1\right)\pi}{2016}}\sin{\frac{\pi}{2016}}=\frac{1}{2}\left\{\cos{\frac{\left(k-1\right)\pi}{1008}}-\cos{\frac{k\pi}{1008}}\right\}
    であるから,
    \sum_{k=1}^{2016}\left\{k\sin{\frac{\left(2k-1\right)\pi}{2016}}\sin{\frac{\pi}{2016}}\right\}=\sum_{k=1}^{2016}\left\{\frac{k}{2}\cos{\frac{\left(k-1\right)\pi}{1008}}-\frac{k}{2}\cos{\frac{k\pi}{1008}}\right\}=\left(\frac{1}{2}\cos{\frac{0}{1008}}-\frac{1}{2}\cos{\frac{\pi}{1008}}\right)+\left(\frac{2}{2}\cos{\frac{\pi}{1008}}-\frac{2}{2}\cos{\frac{2\pi}{1008}}\right)+\left(\frac{3}{2}\cos{\frac{2\pi}{1008}}-\frac{3}{2}\cos{\frac{3\pi}{1008}}\right)+\cdots\cdots+\left(\frac{2016}{2}\cos{\frac{2015\pi}{1008}}-\frac{2016}{2}\cos{\frac{2016\pi}{1008}}\right)=\frac{1}{2}\left(\cos{\frac{0}{1008}}+\cos{\frac{\pi}{1008}}+\cos{\frac{2\pi}{1008}}+\cdots\cdots+\cos{\frac{2015\pi}{1008}}\right)-1008\cos{2\pi}
    S=\cos{\frac{0}{1008}}+\cos{\frac{\pi}{1008}}+\cos{\frac{2\pi}{1008}}+\cdots\cdots+\cos{\frac{2015\pi}{1008}}とおくと,

    上図のように,単位円上では左右対称であるため,項が全て相殺し,0となる.
    \therefore S=0
    よって,
    \sum_{k=1}^{2016}\left\{k\sin{\frac{\left(2k-1\right)\pi}{2016}}\sin{\frac{\pi}{2016}}\right\}=-1008\cos{2\pi}=-1008……(答)

2017年早稲田大学商学部|過去問徹底研究 大問2

2019.09.27

方針の立て方 (1) ガウス記号に関する重要な性質:を使うだけ.ガウス記号は文系数学頻出のテーマのため,この重要な性質とともに覚えておこう. (2) 前問を一般化したもの(前問は本問ののパターンである)であることに気付きたい.入試数学では,まず具体的なパターンでやらせ,その次の問題で一般化するという

  • …続きを読む
  • 方針の立て方

    (1)
    ガウス記号に関する重要な性質:\left[x\right]=n\Leftrightarrow n\leqq x<n+1を使うだけ.ガウス記号は文系数学頻出のテーマのため,この重要な性質とともに覚えておこう.

    (2)
    前問を一般化したもの(前問は本問のp_n=2のパターンである)であることに気付きたい.入試数学では,まず具体的なパターンでやらせ,その次の問題で一般化するという出題形式が多い.一般化されると途端に難しくなったと感じがちだが,前問と同じように処理していけばよい.つまり,\left[x\right]=n\Leftrightarrow n\leqq x<n+1を使って変形し,その範囲で{p_n}^2の倍数であるnを拾い上げていけばよい.ただし,前問ではnに制限がないが,本問では制限がついてしまっていることに注意.

    (3)
    前問の議論で,p_n=1p_n=2,34\leqq p_n\leqq99p_n=100で場合分けしたので,本問でもこれと同様に場合分けして考えればよい.

    解答例

    (1)
    \left[\sqrt[3]{n}\right]=2\Leftrightarrow2\leqq\sqrt[3]{n}<3\Leftrightarrow2^3\leqq n<3^3\Leftrightarrow8\leqq n<27
    である.この範囲で4の倍数となるものが答えである.
    \therefore n=8,12,16,20,24……(答)

    (2)
    nの値に制限がない場合,
    \left[\sqrt[3]{n}\right]=p_n\Leftrightarrow p_n\leqq\sqrt[3]{n}<p_n+1\Leftrightarrow{p_n}^3\leqq n<\left(p_n+1\right)^3\Leftrightarrow{p_n}^3\leqq n<{p_n}^3+3{p_n}^2+3p_n+1
    となる.この範囲に,{p_n}^2の倍数であるnは,
    n={p_n}^3,{p_n}^3+{p_n}^2,{p_n}^3+2{p_n}^2,\cdots\cdots,{p_n}^3+k{p_n}^2
    k+1個ある.ここで,kは,{p_n}^3+k{p_n}^2\leqq{p_n}^3+3{p_n}^2+3p_n\Leftrightarrow k\leqq3+\frac{3}{p_n}を満たす最大の自然数である.つまり,p_n=1ならばk=6,p_n=2,3ならばk=4,p_n\geqq4ならばk=3である.
    今はn\leqq{10}^6という制限があるが,{p_n}^3+3{p_n}^2+3p_n+1\leqq{10}^6\Leftrightarrow\left(p_n+1\right)^3\leqq{10}^6\Leftrightarrow p_n\leqq99までは上記の議論が使える.
    さて,n\leqq{10}^6よりp_n\leqq\left[\sqrt[3]{{10}^6}\right]=\left[100\right]=100であるから,p_n=100のときを別個で考えれば必要十分.
    p_n=100となるnn={10}^6のみであるから,{p_n}^2={100}^2の倍数であるnn={10}^6の1個のみ.
    よって,求める個数は,
    \left(6+1\right)+\left(4+1\right)+\left(4+1\right)+\left(3+1\right)\cdot96+1=402個……(答)

    (3)
    前問の議論より,
    (Ⅰ)p_n=1のとき
    {p_n}^2=1の倍数であるnは,
    n={p_n}^3,{p_n}^3+{p_n}^2,{p_n}^3+2{p_n}^2,\cdots\cdots,{p_n}^3+6{p_n}^2=1,2,3,4,5,6,7
    である.これらをp_n\left(p_n+1\right)=2で割った余りは,順番に1,0,1,0,1,0,1である.
    (Ⅱ)p_n=2のとき
    {p_n}^2=4の倍数であるnは,
    n={p_n}^3,{p_n}^3+{p_n}^2,{p_n}^3+2{p_n}^2,\cdots\cdots,{p_n}^3+4{p_n}^2=8,12,16,20,24
    である.これらをp_n\left(p_n+1\right)=6で割った余りは,順番に2,0,4,2,0である.
    (Ⅲ)p_n=3のとき
    {p_n}^2=9の倍数であるnは,
    n={p_n}^3,{p_n}^3+{p_n}^2,{p_n}^3+2{p_n}^2,\cdots\cdots,{p_n}^3+4{p_n}^2=27,36,45,54,63
    である.これらをp_n\left(p_n+1\right)=12で割った余りは,順番に3,0,9,6,3である.
    (Ⅳ)4\leqq p_n\leqq99のとき
    {p_n}^2の倍数であるnは,
    n={p_n}^3,{p_n}^3+{p_n}^2,{p_n}^3+2{p_n}^2,{p_n}^3+3{p_n}^2
    である.これらをp_n\left(p_n+1\right)で割った余りは,順番にp_n,0,{p_n}^2,{p_n}^2-p_nである.
    (Ⅴ)p_n=100のとき
    {p_n}^2={100}^2の倍数であるnは,
    n={10}^6
    である.これをp_n\left(p_n+1\right)=10100で割った余りは,100である.
    以上,(Ⅰ)~(Ⅴ)より,
    S=\left(1+0+1+0+1+0+1\right)+\left(2+0+4+2+0\right)+\left(3+0+9+6+3\right)+\sum_{p_n=4}^{99}\left\{p_n+0+{p_n}^2+\left({p_n}^2-p_n\right)\right\}+100=656805……(答)

2017年早稲田大学商学部|数学過去問徹底研究 大問3

2019.09.27

方針の立て方 これはチェビシェフ多項式を元に作られた問題である. チェビシェフ多項式は難関大学での三角関数の問題としてよく出される(高等的な数学の知識を必要とせず考察できる)題材であるため,各自調べて,典型問題化しておくと良いだろう. (1) の定義の仕方はでなされているため,をとを用いて表すことを

  • …続きを読む
  • 方針の立て方

    これはチェビシェフ多項式を元に作られた問題である.
    チェビシェフ多項式は難関大学での三角関数の問題としてよく出される(高等的な数学の知識を必要とせず考察できる)題材であるため,各自調べて,典型問題化しておくと良いだろう.

    (1)
    P_n\left(x\right)の定義の仕方はP_n\left(\cos{\theta}\right)でなされているため,P_{n+1}\left(\cos{\theta}\right)P_n\left(\cos{\theta}\right)P_{n-1}\left(\cos{\theta}\right)を用いて表すことを考える.すると,\cos{\left(n+1\right)\theta}\cos{n\theta}\cos{\left(n-1\right)\theta}を用いて表すという問題に帰着する.ただし,最終的にはxに戻さねばならないため,他に使えるのは\cos{\theta}のみである.そのため,途中で出てくる\sin{n\theta}\sin{\theta}\cos{\theta}のみの式となるように変形する.

    (2)
    試しに小さいnをいくつか考えてみるとよい.すると答えの予想がつく.後は前問で漸化式を求めさせていることと,自然数に関する議論であることから,数学的帰納法を用いて,予想が正しいことを示せばよい.

    (3)
    前問ではP_n\left(x\right)x^nのみを特別視して考えていたため,本問もx^nのみを特別視して考えればよいのではと考える.x^n以外の項の解析は難しいが,問題で求められているのが一の位の数字のみであるため,十の位以降に押しやられるのでは直観し,それを示していけばよい.

    解答例

    (1)
    加法定理より,\cos{\left(n+1\right)\theta}=\cos{n\theta}\cos{\theta}-\sin{n\theta}\sin{\theta}
    和積の公式より,
    \sin{n\theta}\sin{\theta}=\frac{1}{2}\left\{\cos{\left(n-1\right)\theta}-\cos{\left(n+1\right)\theta}\right\}
    \therefore\cos{\left(n+1\right)\theta}=\cos{\theta}\cos{n\theta}-\frac{1}{2}\left\{\cos{\left(n-1\right)\theta}-\cos{\left(n+1\right)\theta}\right\}
    整理すると,
    \cos{\left(n+1\right)\theta}=2\cos{\theta}\cos{n\theta}-\cos{\left(n-1\right)\theta}
    \cos{\left(n+1\right)\theta}=P_{n+1}\left(\cos{\theta}\right),\cos{n\theta}=P_n\left(\cos{\theta}\right),\cos{\left(n-1\right)\theta}=P_{n-1}\left(\cos{\theta}\right)であるから,\cos{\theta}=xとすることで,
    P_{n+1}\left(x\right)=2xP_n\left(x\right)-P_{n-1}\left(x\right)……(答)

    (2)
    P_n\left(x\right)x^nの係数が2^{n-1}である(以下ではこの命題を(*)と表す)ことを数学的帰納法で示す.
    n=1のとき,P_1\left(\cos{\theta}\right)=\cos{\theta}よりP_n\left(x\right)=xであるから,(*)は成り立っている.
    n=2のとき,P_2\left(\cos{\theta}\right)=\cos{2\theta}=2\cos^2{\theta}-1よりP_2\left(x\right)=2x^2-1であるから,(*)は成り立っている.
    ここで,n=k,k+1のときの(*)の成立を仮定する.つまり,適当なk-1次以下の多項式Q\left(x\right)と,k次以下の多項式R\left(x\right)とを用いて,
    P_k\left(x\right)=2^{k-1}x^k+Q\left(x\right)
    P_{k+1}\left(x\right)=2^kx^{k+1}+R\left(x\right)
    と書けると仮定する.
    すると,
    P_{k+2}\left(x\right)=2xP_{k+1}\left(x\right)-P_k\left(x\right)=2x\left\{2^kx^{k+1}+R\left(x\right)\right\}-\left\{2^{k-1}x^k+Q\left(x\right)\right\}=2^{k+1}x^{k+2}+\left\{2xR\left(x\right)-2^{k-1}x^k-Q\left(x\right)\right\}
    となる(第一のイコールで(1)で求めた漸化式を,第二のイコールで帰納法の仮定をそれぞれ用いた).
    2xR\left(x\right)-2^{k-1}x^k-Q\left(x\right)は,Q\left(x\right)が高々k-1次,R\left(x\right)が高々k次であるから,高々k+1次である.
    よって,P_{k+2}\left(x\right)x^{k+2}の係数は2^{\left(k+2\right)-1}であると言える.これは,(*)のn=k+2での成立に他ならない.
    以上,数学的帰納法により(*)が示された. 証明終了.
    以上より,求める係数は2^{n-1}……(答)

    (3)
    前問の結果より,
    P_{500}\left(\mathrm{cos}{\theta}\right)=\cos{500\theta}\cos^{500}{\theta}の係数は2^{499}
    よって,\cos{\theta}の499次以下の多項式S\left(\cos{\theta}\right)を用いて,\cos{500\theta}=2^{499}\cos^{500}{\theta}+S\left(\cos{\theta}\right)と表せる.
    よって,\cos{\theta}の999次以下の多項式T\left(\cos{\theta}\right)を用いれば,\cos^2{\left(500\theta\right)}=2^{998}\cos^{1000}{\theta}+T\left(\cos{\theta}\right)と表せる.
    \therefore{10}^{1000}\cos^2{\left(500\theta\right)}={10}^{1000}\cdot2^{998}\cos^{1000}{\theta}+{10}^{1000}T\left(\cos{\theta}\right)
    ここで,{10}^{1000}\cos^{999}{\theta}={10}^{1000}\left(\frac{1}{10}\right)^{999}=10であるから,\cos{\theta}の高々999次の多項式であるT\left(\cos{\theta}\right){10}^{1000}をかけた{10}^{1000}T\left(\cos{\theta}\right)は一の位の数に寄与しない.
    よって,{10}^{1000}\cos^2{\left(500\theta\right)}の一の位の数は{10}^{1000}\cdot2^{998}\cos^{1000}{\theta}={10}^{1000}\cdot2^{998}\left(\frac{1}{10}\right)^{1000}=2^{998}と等しくなる.

    n 1 2 3 4 5 6 7 8 9 10 11 12 13 \cdots
    2^nの一の位の数 2 4 8 6 2 4 8 6 2 4 8 6 2 \cdots

    上表のように,2^nの一の位の数は2,4,8,6が繰り返される.これを用いると2^{998}の一の位の数は4と分かる.
    よって,求める数は4……(答)

2017年早稲田大学商学部|過去問徹底研究 大問1

2019.09.25

方針の立て方 (1) 絶対値問題の典型的解法,つまり,場合分けをして絶対値記号を外すことを試みる.その後は二次関数の最大最小問題と同じように,区間とグラフの位置関係で場合分けを行う.場合分けのパターンが多いが,対称性があるため⑤~⑦は実質的に計算しなくても答えは分かる.後はと直線を図示して面積を求め

  • …続きを読む
  • 方針の立て方
    (1)
    絶対値問題の典型的解法,つまり,場合分けをして絶対値記号を外すことを試みる.その後は二次関数の最大最小問題と同じように,区間とグラフの位置関係で場合分けを行う.場合分けのパターンが多いが,対称性があるため⑤~⑦は実質的に計算しなくても答えは分かる.後はf\left(x\right)と直線y=1を図示して面積を求めるのみ.

    (2)
    4次方程式の解析は難しいため,次数を下げることを考える.そこで「x=\alphaは代数方程式P\left(x\right)=0の解である」⇔「多項式P\left(x\right)x-\alphaを因数にもつ」という解の重要性質を利用すると考えよう.この重要性質を使えば,2次方程式の解析問題に帰着させられる.後は,実数解なのか虚数解なのかで場合分けをして考えればよい.

    (3)
    長さの問題であるため,座標系を導入すると考えやすくなると考える.「座標は長さの問題のときに強く,角度の問題のときには弱い」というのは覚えておこう.後は問題の状況を丁寧に書き下していけばよい.平方完成を用いた最小値問題は頻出問題なのでおさえておくこと.

    (4)
    {10}^{-k}2^n{10}^{100}3^{-n}のどちらが\mathrm{max}\left\{{10}^{-k}2^n,{10}^{100}3^{-n}\right\}の値になるかを考えよう(絶対値記号と同様に\mathrm{max}もそのままでは扱いにくいので外すことをまず考える).「全ての整数nに対して」となっているので,まずはkを固定してnのみを変数扱いして考えよう.{10}^{-k}2^n{10}^{100}3^{-n}はそれぞれ単調増加,単調減少であるため,最初は{10}^{-k}2^n<{10}^{100}3^{-n}となるだろうと分かる.そこで{10}^{-k}2^n={10}^{100}3^{-n}となるnを考える.後は十分条件を考え,そのあとで,必要性を考える.つまり,k\leqq63 \Rightarrow \mathrm{max}\left\{{10}^{-k}2^n,{10}^{100}3^{-n}\right\}\geqq1は言えるが,ではkをこれより大きくした場合はどうか,具体的にはk=64,65,66,\cdots\cdotsはどうかを考える必要があると考える.するとk=64で(*)を満たさないことが確認できるので,答えは63と分かる.

    解答例
    (1)
    ア:\frac{5}{3}
    (2)
    イ:-3
    ウ:-6
    (3)
    エ:\frac{6}{5}
    (4)
    オ:63

    解説
    (1)
    g\left(t\right)=\left|\left|t\right|-1\right|とおくと,

    ①のとき(x+1\leqq-1\Leftrightarrow x\leqq-2)
    f\left(x\right)=\frac{1}{2}\int_{x-1}^{x+1}\left(-t-1\right)dt=\frac{1}{2}\left[-\frac{1}{2}t^2-t\right]_{x-1}^{x+1}=-x-1
    ②のとき(-1\leqq x+1\leqq0\Leftrightarrow-2\leqq x\leqq-1)
    f\left(x\right)=\frac{1}{2}\int_{x-1}^{-1}\left(-t-1\right)dt+\frac{1}{2}\int_{-1}^{x+1}\left(t+1\right)dt=\frac{1}{2}\left[-\frac{1}{2}t^2-t\right]_{x-1}^{-1}+\frac{1}{2}\left[\frac{1}{2}t^2+t\right]_{-1}^{x+1}=\frac{1}{2}\left(x^2+2x+2\right)
    ③のとき(0\leqq x+1\leqq1\Leftrightarrow-1\leqq x\leqq0)
    f\left(x\right)=\frac{1}{2}\int_{x-1}^{-1}\left(-t-1\right)dt+\frac{1}{2}\int_{-1}^{0}\left(t+1\right)dt+\frac{1}{2}\int_{0}^{x+1}\left(-t+1\right)dt=\frac{1}{2}\left[-\frac{1}{2}t^2-t\right]_{x-1}^{-1}+\frac{1}{2}\left[\frac{1}{2}t^2+t\right]_{-1}^0+\frac{1}{2}\left[-\frac{1}{2}t^2+t\right]_0^{x+1}=\frac{1}{2}
    ④のとき(x+1=1\Leftrightarrow x=0)
    f\left(x\right)=\frac{1}{2}\int_{-1}^{0}\left(t+1\right)dt+\frac{1}{2}\int_{0}^{1}\left(-t+1\right)dt=\frac{1}{2}\left[\frac{1}{2}t^2+t\right]_{-1}^0+\frac{1}{2}\left[-\frac{1}{2}t^2+t\right]_0^1=\frac{1}{2}
    ⑤のとき(-1\leqq x -1\leqq 0 \Leftrightarrow 0\leqq x\leqq1)
    f\left(x\right)=\frac{1}{2}\int_{x-1}^{0}\left(t+1\right)dt+\frac{1}{2}\int_{0}^{1}\left(-t+1\right)dt+\frac{1}{2}\int_{1}^{x+1}\left(t-1\right)dt=\frac{1}{2}\left[\frac{1}{2}t^2+t\right]_{x-1}^0+\frac{1}{2}\left[-\frac{1}{2}t^2+t\right]_0^1+\frac{1}{2}\left[\frac{1}{2}t^2-t\right]_1^{x+1}=\frac{1}{2}
    ⑥のとき(0\leqq x-1\leqq1\Leftrightarrow1\leqq x\leqq2)
    f\left(x\right)=\frac{1}{2}\int_{x-1}^{1}\left(-t+1\right)dt+\frac{1}{2}\int_{1}^{x+1}\left(t-1\right)dt=\frac{1}{2}\left[-\frac{1}{2}t^2+t\right]_{x-1}^1+\frac{1}{2}\left[\frac{1}{2}t^2-t\right]_1^{x+1}=\frac{1}{2}\left(x^2-2x+2\right)
    ⑦のとき(1\leqq x-1\Leftrightarrow2\leqq x)
    f\left(x\right)=\frac{1}{2}\int_{x-1}^{x+1}\left(t-1\right)dt=\frac{1}{2}\left[\frac{1}{2}t^2-t\right]_{x-1}^{x+1}=x-1
    まとめると,
    f\left(x\right)=\begin{cases} -x-1 \left(x\leqq-2\right) \\ \frac{1}{2}\left(x^2+2x+2\right) \left(-2\leqq x\leqq-1\right) \\ \frac{1}{2} \left(-1\leqq x\leqq1\right) \\ \frac{1}{2}\left(x^2-2x+2\right) \left(1\leqq x\leqq2\right) \\ x-1 \left(2\leqq x\right) \end{cases}
    図示すると,

    よって,求める面積は,y軸での対称性より,
    2\left\{\frac{1}{2}\cdot1+\int_{1}^{2}\left\{1-\frac{1}{2}\left(x^2-2x+2\right)\right\}dx\right\}=1+2\left[-\frac{1}{6}x^3+\frac{1}{2}x^2\right]_1^2=\frac{5}{3}……(答)

    (2)
    実数解が1と3であることから,他の二解をx=\alpha,\betaとして,
    x^4+ax^3+bx^2+cx+3=\left(x-1\right)\left(x-3\right)\left(x-\alpha\right)\left(x-\beta\right)=x^4-\left(\alpha+\beta+4\right)x^3+\left(4\alpha+4\beta+\alpha\beta+3\right)x^2-\left(4\alpha\beta+3\alpha+3\beta\right)x+3\alpha\beta
    と書ける.係数比較すると,
    \begin{cases} a=-\left(\alpha+\beta+4\right) \\ b=4\alpha+4\beta+\alpha\beta+3 \\ c=-\left(4\alpha\beta+3\alpha+3\beta\right) \\ 3=3\alpha\beta \end{cases}\Leftrightarrow\begin{cases} \alpha+\beta=-a-4 \\ 4\alpha+4\beta=b-4 \\ 3\alpha+3\beta=-c-4 \\ \alpha\beta=1 \end{cases}
    となる.
    次に,2次方程式\left(x-\alpha\right)\left(x-\beta\right)=0\Leftrightarrow x^2-\left(\alpha+\beta\right)x+\alpha\beta=0について考える.この方程式の解が1か3,或いは虚数解であれば,4次方程式x^4+ax^3+bx^2+cx+3=0の実数解が1と3のみとなる.
    (Ⅰ)\alpha,\betaが実数のとき
    まず,判別式が非負となる必要があるので,\left(\alpha+\beta\right)^2-4\cdot1\cdot\alpha\beta\geqq0\Leftrightarrow\left(\alpha-\beta\right)^2\geqq 0が必要である.
    このもとで,2次方程式x^2-\left(\alpha+\beta\right)x+\alpha\beta=0の解が1か3のみとなるには,\left(\alpha,\beta\right)=\left(1,1\right),\left(1,3\right),\left(3,1\right),\left(3,3\right)なら必要十分(これらは全て\left(\alpha-\beta\right)^2\geqq0を満たす).この内,(*)式に抵触しないのは,\left(\alpha,\beta\right)=\left(1,1\right)のみである.このとき,(*)の第一式より,a=-6となる.
    (Ⅱ)\alpha,\betaが虚数のとき
    まず,判別式が負となる必要があるので,\left(\alpha+\beta\right)^2-4\cdot1\cdot\alpha\beta<0\Leftrightarrow\left(\alpha-\beta\right)^2<0が必要である.
    \alpha,\betaが虚数ならば,\alpha,\betaの値によらず,2次方程式x^2-\left(\alpha+\beta\right)x+\alpha\beta=0の解は虚数となる.
    (*)を利用すれば,\left(\alpha-\beta\right)^2<0\Leftrightarrow\left(\alpha+\beta\right)^2-4\alpha\beta<0\Leftrightarrow\left(-a-4\right)^2<4\Leftrightarrow-6<a<-2
    以上(Ⅰ)と(Ⅱ)より,4次方程式x^4+ax^3+bx^2+cx+3=0の実数解が1と3のみとなるaの範囲は-6\leqq a<-2である.
    aは整数なので,求める最大値は-3,最小値は-6である.……(答)

    (3)
    {\mathrm{AB}}^2+{\mathrm{BC}}^2={\mathrm{CA}}^2より,三角形\mathrm{ABC}\angle\mathrm{B}={90}^\circの直角三角形である.

    そこで,点\mathrm{B}を原点として,左図のように三角形\mathrm{ABC}xy平面上にのせる.
    内部の点\mathrm{O}の座標を左図のように\left(X,Y\right)とおく.点\mathrm{O}は三角形\mathrm{ABC}の内部の点であるので,
    \begin{cases} 0\leqq X \\ 0\leqq Y \\ Y\leqq-\frac{3}{4}X+3 \end{cases}……(*)
    を満たす必要がある.
    このもとで,
    {\mathrm{OP}}^2=X^2,{\mathrm{OQ}}^2=Y^2
    である.更に点と直線の距離の公式より,
    {\mathrm{OR}}^2=\frac{\left(3X+4Y-12\right)^2}{3^2+4^2}=\frac{9X^2+16Y^2-72X-96Y+24XY+144}{25}
    である.
    \therefore{\mathrm{OP}}^2+{\mathrm{OQ}}^2+{\mathrm{OR}}^2=X^2+Y^2+\frac{9X^2+16Y^2-72X-96Y+24XY+144}{25}=\frac{1}{25}\left\{34\left(X+\frac{6Y-18}{17}\right)^2+\frac{1}{17}\left(25Y-24\right)^2+72\right\}
    よって,
    \begin{cases} X+\frac{6Y-18}{17}=0 \\ 25Y-24=0 \end{cases}\Leftrightarrow\begin{cases} X=\frac{18}{25} \\ Y=\frac{24}{25} \end{cases}\mathrm{OP}^2+\mathrm{OQ}^2+\mathrm{OR}^2は最小となる.なお,\begin{cases} X=\frac{18}{25} \\ Y=\frac{24}{25} \end{cases}は(*)を満たす.
    このとき,
    \mathrm{OR}=\frac{\left|3\cdot\frac{18}{25}+4\cdot\frac{24}{25}-12\right|}{5}=\frac{6}{5}……(答)

    (4)
    kを固定して,{10}^{-k}2^n={10}^{100}3^{-n}となるnについて考えると,{10}^{-k}2^n={10}^{100}3^{-n}\Leftrightarrow6^n={10}^{100+k}より,n=\log_6{{10}^{100+k}}=\left(100+k\right)\log_6{10}=\frac{100+k}{\log_{10}{6}}=\frac{100+k}{\log_{10}{2}+\log_{10}{3}}
    {10}^{-k}2^nnについて単調増加であり,{10}^{100}3^{-n}は単調減少であるから,\mathrm{max}\left\{{10}^{-k}2^n,{10}^{100}3^{-n}\right\}の最小値は,{10}^{100}3^{-\frac{100+k}{\log_{10}{2}+\log_{10}{3}}}以上である.
    よって,\mathrm{max}\left\{{10}^{-k}2^n,{10}^{100}3^{-n}\right\}\geqq 1を満たすには,
    {10}^{100}3^{-\frac{100+k}{{\mathrm{log}}_{\mathrm{10}}{\mathrm{2}}\mathrm{+} {\mathrm{log}}_{\mathrm{10}}{\mathrm{3}}}}\geqq1\Leftrightarrow{10}^{100}\geqq\mathrm{3}^\frac{100+k}{{\mathrm{log}}_{\mathrm{10}}{\mathrm{2}}\mathrm{+} {\mathrm{log}}_{\mathrm{10}}{\mathrm{3}}}\Leftrightarrow100\geqq\frac{100+k}{{\mathrm{log}}_{\mathrm{10}}{\mathrm{2}}\mathrm{+} {\mathrm{log}}_{\mathrm{10}}{\mathrm{3}}}{\mathrm{log}}_{\mathrm{10}}{\mathrm{3}}\Leftrightarrow k\leqq100\frac{{\mathrm{log}}_{\mathrm{10}}{\mathrm{2}}}{{\mathrm{log}}_{\mathrm{10}}{\mathrm{3}}}
    であれば十分.
    100\frac{{\mathrm{log}}_{\mathrm{10}}{\mathrm{2}}}{{\mathrm{log}}_{\mathrm{10}}{\mathrm{3}}}>100\frac{0.301}{0.4772}=63.07,100\frac{{\mathrm{log}}_{\mathrm{10}}{\mathrm{2}}}{{\mathrm{log}}_{\mathrm{10}}{\mathrm{3}}}<100\frac{0.3011}{0.4771}=63.11kが整数であることから,
    k\leqq63であれば十分.
    ここで,k=64のときを考える.
    \frac{100+k}{\log_{10}{2}+\log_{10}{3}}=\frac{164}{\log_{10}{2}+\log_{10}{3}}であり\frac{164}{0.3011+0.4772}<\frac{164}{\log_{10}{2}+\log_{10}{3}}<\frac{164}{0.301+0.4771}\Leftrightarrow210.71\mathrm{\cdots\cdots}<\frac{164}{\log_{10}{2}+\log_{10}{3}}<210.76\mathrm{\cdots\cdots}より,\mathrm{max}\left\{{10}^{-64}2^n,{10}^{100}3^{-n}\right\}の最小値は,{10}^{100}3^{-210}{10}^{-64}2^{211}である.
    \log_{10}{\left({10}^{100}3^{-210}\right)}=100-210\log_{10}{3}<100-210\cdot0.4771=-0.191
    \log_{10}{\left({10}^{-64}2^{211}\right)}=-64+211\log_{10}{2}<-64+211\cdot0.3011=-0.4679
    より,{10}^{100}3^{-210}<{10}^{-0.191}<{10}^0=1,{10}^{-64}2^{211}<{10}^{-0.4679}<{10}^0=1であるから,k=64のとき条件(*)は満たされない.
    よって求めるkの最大値は63……(答)

2018年早稲田大学商学部|過去問徹底研究 大問3

2019.09.24

方針の立て方 (1) 実際にを求めていくことで解答を得られる. (2) 前問での議論で,には周期性があることが分かる.更に,大事なのはとのなす角であることも分かるだろう(もし前問の議論だけでは方針を得られない場合には,他の具体的な組み合わせで考えてみると良い).そこで,とのなす角で場合分けをして議論

  • …続きを読む
  • 方針の立て方
    (1)
    実際に\mathrm{A}_nを求めていくことで解答を得られる.

    (2)
    前問での議論で,\mathrm{A}_nには周期性があることが分かる.更に,大事なのは\mathrm{O}\mathrm{A}_1\mathrm{O}\mathrm{A}_2のなす角であることも分かるだろう(もし前問の議論だけでは方針を得られない場合には,他の具体的な組み合わせで考えてみると良い).そこで,\mathrm{O}\mathrm{A}_1\mathrm{O}\mathrm{A}_2のなす角で場合分けをして議論していけば良いと判断する.

    解答例
    (1)

    (ⅰ)と(ⅱ)に従って\mathrm{A}_nを求めていくと,上図のようになる.
    上図より\mathrm{A}_{15}=\mathrm{P}_3であるから,求めるkk=3……(答)

    (2)
    \mathrm{A}_n=\mathrm{P}_kとして,前問の議論(\left(\mathrm{A}_1,\mathrm{A}_2\right)=\left(\mathrm{P}_1,\mathrm{P}_2\right)のとき)をまとめると,下表のようになる.

    n 1 2 3 4 5 6 7 8 9 10 11 12 13 14 15 \cdots
    k 1 2 9 4 5 3 7 8 6 1 2 9 4 5 3 \cdots

    これより,kの値は1,2,9,4,5,3,7,8,6という周期9の並びを繰り返すことが分かる.
    \mathrm{A}_n=\mathrm{P}_1となるnが存在するため\left(\mathrm{A}_1,\mathrm{A}_2\right)=\left(\mathrm{P}_1,\mathrm{P}_2\right)は題意を満たさない.
    以下,\left(\mathrm{A}_1,\mathrm{A}_2\right)=\left(\mathrm{P}_i,\mathrm{P}_j\right)として,i<jのみを考える.更に\mathrm{O}\mathrm{P}_i\mathrm{O}\mathrm{P}_jのなす角の内,小さい方を\theta_{ij}と表す.
    (Ⅰ)\theta_{ij}=\frac{2\pi}{9}となるi,jのとき
    実際に\mathrm{A}_nを求めていくと,\left(\mathrm{A}_1,\mathrm{A}_2\right)=\left(\mathrm{P}_1,\mathrm{P}_2\right)のときのようにkの値は周期9の並びを繰り返し,kは1から9の全ての値をとる.よって,題意を満たさない.
    (Ⅱ)\theta_{ij}=\frac{4\pi}{9}となるi,jのとき
    実際に\mathrm{A}_nを求めていく.例えば\left(\mathrm{A}_1,\mathrm{A}_2\right)=\left(\mathrm{P}_1,\mathrm{P}_3\right)の場合,

    上図のようになる.
    \mathrm{A}_n=\mathrm{P}_kとしてまとめると,下表のようになる.

    n 1 2 3 4 5 6 7 8 9 10 11 12 \cdots
    k 1 3 8 7 9 5 4 6 2 1 3 8 \cdots

    これより,kの値は1,3,8,7,9,5,4,6,2という周期9の並びを繰り返すことが分かる.
    \mathrm{A}_n=\mathrm{P}_1となるnが存在するため\left(\mathrm{A}_1,\mathrm{A}_2\right)=\left(\mathrm{P}_1,\mathrm{P}_3\right)は題意を満たさない.また,他のi,jについても同様に題意を満たさない.
    (Ⅲ)\theta_{ij}=\frac{6\pi}{9}となるi,jのとき
    実際に\mathrm{A}_nを求めていく.例えば\left(\mathrm{A}_1,\mathrm{A}_2\right)=\left(\mathrm{P}_1,\mathrm{P}_4\right)の場合,

    上図のようになる.
    \mathrm{A}_n=\mathrm{P}_kとしてまとめると,下表のようになる.

    n 1 2 3 4 5 6 \cdots
    k 1 4 7 1 4 7 \cdots

    これより,kの値は1,4,7という周期3の並びを繰り返すことが分かる.
    \mathrm{A}_n=\mathrm{P}_1となるnが存在しないため\left(\mathrm{A}_1,\mathrm{A}_2\right)=\left(\mathrm{P}_1,\mathrm{P}_4\right)は題意を満たす.また,他のi,jについても同様に題意を満たす.
    \theta_{ij}=\frac{6\pi}{9}となるi,jの組み合わせは\left(i,j\right)=\left(2,5\right),\left(2,8\right),\left(3,6\right),\left(3,9\right),\left(5,8\right),\left(6,9\right)であり,これら6組は題意を満たす.
    (Ⅳ)\theta_{ij}=\frac{8\pi}{9}となるi,jのとき
    実際に\mathrm{A}_nを求めていく.例えば\left(\mathrm{A}_1,\mathrm{A}_2\right)=\left(\mathrm{P}_1,\mathrm{P}_5\right)の場合,

    n 1 2 3 4 5 6 7 8 9 10 11 12 \cdots
    k 1 5 9 4 8 3 7 2 6 1 5 9 \cdots

    これより,kの値は1,5,9,4,8,3,7,2,6という周期9の並びを繰り返すことが分かる.
    \mathrm{A}_n=\mathrm{P}_1となるnが存在するため\left(\mathrm{A}_1,\mathrm{A}_2\right)=\left(\mathrm{P}_1,\mathrm{P}_5\right)は題意を満たさない.また,他のi,jについても同様に題意を満たさない.
    以上,(Ⅰ)~(Ⅳ)より,題意を満たすi,jの組み合わせはi>jの範囲でも題意を満たすi,jの組み合わせは6組あるので,求める個数は6+6=12個……(答)

2018年早稲田大学商学部|過去問徹底研究 大問2

2019.09.24

方針の立て方 (1) 試しにを書き下すと解答が得られる.このときに分母を2で割った値が大事になることや,分母が奇数のときにはもう議論を続ける必要がないことが分かるだろう. (2) 前問の議論を一般化して考える.前問の議論で,分母が偶数であるときには,その分母の数字を2で割った値が大事になり,分母が奇

  • …続きを読む
  • 方針の立て方
    (1)
    試しにa_1\left(\frac{i}{12}\right),a_2\left(\frac{i}{12}\right),a_3\left(\frac{i}{12}\right),\cdots\cdotsを書き下すと解答が得られる.このときに分母を2で割った値が大事になることや,分母が奇数のときにはもう議論を続ける必要がないことが分かるだろう.

    (2)
    前問の議論を一般化して考える.前問の議論で,分母が偶数であるときには,その分母の数字を2で割った値が大事になり,分母が奇数になったときに議論が終了することから,xに素因数2が何個含まれているかがカギになると見抜きたい.後は前問のように場合分けして考えていくことを考えれば,解答が得られる.

    解答例
    (1)
    i=1,2,\cdots\cdots,11として,
    a_1\left(\frac{i}{12}\right)=\frac{i}{12}\neq0


    ここで,a_4\left(\frac{i}{12}\right)について考えると,

    となる.ここで,\frac{2i}{3},\frac{2\left(i-3\right)}{3},\frac{2\left(i-6\right)}{3},\frac{2\left(i-9\right)}{3}は全て整数とはならない.一方で\left[\frac{2i}{3}\right],\left[\frac{2\left(i-3\right)}{3}\right],\left[\frac{2\left(i-6\right)}{3}\right],\left[\frac{2\left(i-9\right)}{3}\right]は全て整数である.よって,\frac{2i}{3}-\left[\frac{2i}{3}\right],\frac{2\left(i-3\right)}{3}-\left[\frac{2\left(i-3\right)}{3}\right],\frac{2\left(i-6\right)}{3}-\left[\frac{2\left(i-6\right)}{3}\right],\frac{2\left(i-9\right)}{3}-\left[\frac{2\left(i-9\right)}{3}\right]は全て0とはならない.
    同様に,a_5\left(\frac{i}{12}\right),a_6\left(\frac{i}{12}\right),\cdots\cdotsでもi=1,2,4,5,7,8,10,11のときは0とはならない.
    よって,i=3,6,9のみが(*)を満たす.
    \therefore S_{12}=\left\{\frac{1}{4},\frac{1}{2},\frac{3}{4}\right\}……(答)

    (2)
    前問の議論を応用すれば,xが有理数で分母が偶数(ある自然数mを用いて2mと表す)であるときa_2\left(x\right)i=mで0となる.その後はi=1,2,\cdots\cdots,m-1i=m+1,m+2,\cdots\cdots,n-1で場合分けして同様の議論が繰り返せる.この議論は,a_k\left(x\right)の分母が奇数となるまで続く.
    よって,xが有理数で分母を素因数分解したときに2^l(lは0以上の整数)を含む場合,a_2\left(x\right)=0となるiは1個あり,a_3\left(x\right)=0となるiは(a_2\left(x\right)=0となるiを除くと)2個あり,a_4\left(x\right)=0となるiは(a_3\left(x\right)=0となるiを除くと)4個あり,……,a_{l+1}\left(x\right)=0となるiは(a_l\left(x\right)=0となるiを除くと)2^{l-1}個ある.なお,a_{l+2}\left(x\right)=0となるia_{l+1}\left(x\right)=0となるiを除くと存在しない.
    よって,(*)を満たすi\sum_{k=0}^{k=l-1}2^k=2^l-1個存在する.
    そして(*)を満たす有理数は,\frac{i}{2^l}(i=1,2,\cdots\cdots,2^l-1)である.
    よって,Tの要素の個数は,1から2018の中で素因数に2を最も多く含むもののを考え,その数の素因数2の個数をm個とすれば,2^m-1個である.
    2^m\leqq2018を満たす最大のmm=10である.
    よって求める個数は,
    2^{10}-1=1023個……(答)

2018年早稲田大学商学部|過去問徹底研究 大問1

2019.09.24

方針の立て方 (1) まずは,扱いにくい絶対値記号を外す.の正負で場合分けを行えばよい. 絶対値を外せば,方程式は1次方程式になる.方程式のまま解析しても良いが,「方程式の解に関する解析は方程式の左辺(0でない方)を関数でおいて軸との交点で考える」という王道手段を本解では採用した. (2) 整数問題

  • …続きを読む
  • 方針の立て方
    (1)
    まずは,扱いにくい絶対値記号を外す.x-1の正負で場合分けを行えばよい.
    絶対値を外せば,方程式は1次方程式になる.方程式のまま解析しても良いが,「方程式の解に関する解析は方程式の左辺(0でない方)を関数でおいてx軸との交点で考える」という王道手段を本解では採用した.

    (2)
    整数問題の典型問題である.素数の累乗のため約数に持ち込む(積の形に持ち込む)と都合が良いと考え因数分解を行う.

    (3)
    P\left(x\right)が整式である」という情報をどう盛り込むかを考える.できることならP\left(x\right)を具体的に書き下したいが,その際に次数が分かっていないのがネックになるため,まずは次数を求めることに専念する.次数が求まれば,後は具体的にP\left(x\right)を書き下して,計算するのみ.

    (4)
    このような抽象的な関数の問題では,数式の意味を考えると良い.例えばf\left(-x\right)=-f\left(x\right)は「引数の符号を反転させると,関数値の符号が反転する」ことを意味していると考える.すると,1-xの符号を反転させれば,f\left(1+x\right)=f\left(1-x\right)は引数xの係数の符号が揃い,f\left(x+m\right)=f\left(x\right)に近づくと考える.
    次にf\left(x+1\right)=-f\left(x-1\right)は「引数が2上下すると,関数値の符号が反転する」ことを意味していると考える.すると「引数が4上下すると,関数値の符号は同じになる(2回反転して元に戻る)」と分かり,答えにたどり着く.解答では,この当たりを厳密に数式で処理しているが,本番では途中経過を求められないで,このような定性的な議論で十分だろう.

    解答例
    (1)ア:\frac{-1+\sqrt{13}}{2}
    (2)イ:\left(17,2,6\right)
    (3)ウ:3x
    (4)エ:4

    解説
    (1)
    x\geqq1のとき
    方程式は,
    \left(1-a\right)x+k^2+ak-3=0
    となる.ここで,f_1\left(x\right)=\left(1-a\right)x+k^2+ak-3とおく.
    x<1のとき
    方程式は,
    \left(-1-a\right)x+k^2+ak-1=0
    となる.ここで,f_2\left(x\right)=\left(-1-a\right)x+k^2+ak-1とおく.
    さらに,
    g\left(x\right)=\begin{cases} f_1\left(x\right)\ \left(x\geqq1\right) \\ f_2\left(x\right)\ \left(x<1\right) \end{cases}
    とおく.ここで,g\left(x\right)x=1で連続であることに注意.
    (Ⅰ)\begin{cases} 0<1-a \\ 0<-1-a \end{cases}\Leftrightarrow a<-1のとき
    関数y=f_1\left(x\right)y=f_2\left(x\right)も傾き正の一次関数であるから,g\left(x\right)-\inftyから+\inftyの値を取り得る.よって,kの値によらずg\left(x\right)=0となるxは存在する.
    (Ⅱ)\begin{cases} 0>1-a \\ 0>-1-a \end{cases}\Leftrightarrow 1<aのとき
    関数y=f_1\left(x\right)y=f_2\left(x\right)も傾き負の一次関数であるから,g\left(x\right)-\inftyから+\inftyの値を取り得る.よって,kの値によらずg\left(x\right)=0となるxは存在する.
    (Ⅲ)\begin{cases} 0\leqq1-a \\ 0\geqq-1-a \end{cases}\Leftrightarrow-1\leqq a\leqq1のとき
    関数y=f_1\left(x\right)は傾き0以上の一次関数で,関数y=f_2\left(x\right)は傾き0以下の一次関数である.よって,g\left(x\right)の最小値はx=1のときでg\left(1\right)=k^2+ak-2-aである.なお最大値は存在しない.
    よってaの値に依らず解が存在するには全てのaに対してg\left(1\right)\leqq0であれば必要十分.
    g\left(1\right)\leqq0\Leftrightarrow k^2+ak-2-a\leqq0\Leftrightarrow\left(k-1\right)a+k^2-2\leqq0……(*)
    -1\leqq a\leqq1に気を付けると,

    となるから,(*)の条件式は,

    となる.よって求める最大値は\frac{-1+\sqrt{13}}{2}……(答)

    (2)
    225=3^2\cdot5^2={15}^2より,
    a^2=b^n+225\Leftrightarrow\left(a-15\right)\left(a+15\right)=b^n
    となる.この式より,a-15a+15b^nの約数となることが分かる.また,bは素数であることから,b^nの約数は1,b,b^2,\cdots\cdots,b^nである.よって,
    \begin{cases} a-15=b^k \\ a+15=b^{n-k} \end{cases}
    と書ける.ここで,kは0以上の整数であり,a-15<a+15よりk<n-k\Leftrightarrow2k<nを満たす.
    両辺の差を取ると,
    30=b^{n-k}-b^k=b^k\left(b^{n-2k}-1\right)
    となる.この式より,b^kb^{n-2k}-1は30の約数となることが分かるが,bが素数であることを加味すれば,b^kb^{n-2k}-1の考えられる組み合わせは
    \left(b^k,b^{n-2k}-1\right)=\left(1,30\right),\left(2,15\right),\left(3,10\right),\left(5,6\right)
    の4つ.この内,整合性が取れるのは,\left(b^k,b^{n-2k}-1\right)=\left(2,15\right)のみであり,解くと,
    \left(b,k,n\right)=\left(2,1,6\right)
    となる.これをa-15=b^kに代入すれば,a=17と分かる.
    \therefore\left(a,b,n\right)=\left(17,2,6\right)……(答)

    (3)
    P\left(x\right)n次の多項式(nは自然数)とすると,(左辺)=\int_{0}^{x}\left\{P\left(t\right)\right\}^mdtnm+1次の多項式となる.
    一方で,(右辺)=P\left(x^3\right)-P\left(0\right)3nの多項式である.
    左辺と右辺の次数は等しいため,
    nm+1=3n\Leftrightarrow n=\frac{1}{3-m}
    となる.nが自然数であるため\frac{1}{3-m}も自然数であり,m=2であれば必要十分.また,そのときn=1である.
    よって,P\left(x\right)は1次多項式であるから,0でない実数aと実数bを用いて,
    P\left(x\right)=ax+b
    と表せる.
    \int_{0}^{x}\left\{P\left(t\right)\right\}^mdt=\int_{0}^{x}\left\{at+b\right\}^2dt=\left[\frac{1}{3}a^2t^3+abt^2+b^2t\right]_0^x=\frac{1}{3}a^2x^3+abx^2+b^2x P\left(x^3\right)-P\left(0\right)=\left(ax^3+b\right)-b=ax^3
    より,両辺の係数比較をすると,a\neq0に注意して,
    \begin{cases} \frac{1}{3}a^2=a \\ ab=0 \\ b^2=0 \end{cases}\Leftrightarrow\begin{cases} a=3 \\ b=0 \end{cases}
    \therefore P\left(x\right)=3x

    (4)
    f\left(-x\right)=-f\left(x\right)\Leftrightarrow f\left(x\right)=-f\left(-x\right)で,x1-xを代入すると,
    f\left(1-x\right)=-f\left(x-1\right)
    が言える.
    \therefore f\left(1+x\right)=f\left(1-x\right)\Leftrightarrow f\left(x+1\right)=-f\left(x-1\right)……(*)
    更に,(*)でxx-2を代入すると,
    f\left(x-1\right)=-f\left(x-3\right)
    となるから,(*)の右辺に代入すると
    f\left(x+1\right)=f\left(x-3\right)
    さらに,この式でxx+3を代入すると,
    f\left(x+4\right)=f\left(x\right)
    となる.よって,求めるmの最小値は4……(答)

2016年慶應義塾大学環境情報|過去問徹底研究 大問6

2019.09.23

方針の立て方 (1)(2)ともに,ケース1~4のどのケースが適用されるのかが直観的に分からないため,「仮にこのケースが適用されるなら」と考えて,ケース1から順番に代入していく.このような既存の分野にとらわれない新傾向の出題はSFCや商学部で多く見られるが,これら新傾向問題の攻略法は地道に片っ端から試

  • …続きを読む
  • 方針の立て方
    (1)(2)ともに,ケース1~4のどのケースが適用されるのかが直観的に分からないため,「仮にこのケースが適用されるなら」と考えて,ケース1から順番に代入していく.このような既存の分野にとらわれない新傾向の出題はSFCや商学部で多く見られるが,これら新傾向問題の攻略法は地道に片っ端から試してみることにある.そのまま代入したり試行したりすることで答えまで至る今回のような問題もあれば,途中で規則性に気付いて解答する問題もある.どちらにせよ,分からなかったら試してみるということを心がけよう.

    解答例
    (85)(86)(87)……060
    (88)(89)(90)……180
    (91)(92)(93)……150
    (94)(95)(96)……200
    (97)(98)(99)……035
    (100)(101)(102)……035
    (103)(104)(105)……050
    (106)(107)(108)……140

    解説
    (1)
    Aの範囲((85)~(90)について)
    ケース1が適用されるなら,A\leqq\frac{2}{2}\cdot60=60が必要で,X_1=30となるには,A=60が必要.
    ケース2が適用されるなら,k=1となるから,\frac{1}{2}\cdot240-\frac{1}{2}\cdot120\leqq A\leqq\frac{1}{2}\cdot240-\frac{1}{2}\cdot0\Leftrightarrow60\leqq A\leqq120のときX_1=\frac{1}{2}B_1=30となるため,60\leqq A\leqq120が必要となる.
    ケース3が適用されるなら,k=1となるから,\frac{1}{2}\cdot240+\frac{1}{2}\cdot0\leqq A\leqq\frac{1}{2}\cdot240+\frac{1}{2}\cdot120\Leftrightarrow120\leqq A\leqq180のときX_1=\frac{1}{2}B_1=30となるため,120\leqq A\leqq180が必要となる.
    ケース4が適用されるなら,240-\frac{2}{2}\cdot60\leqq A\Leftrightarrow180\leqq AのときX_1=60-\frac{1}{2}\left(240-A\right)となるため,X_1=30となるにはA=180が必要となる.
    以上より,60\leqq A\leqq180……(答)
    X_2X_1の4倍となるとき((91)~(96)について)
    ケース1が適用されるなら,X_1=X_2=\frac{A}{2}より,満たすAは存在しない.
    ケース2が適用されるなら,k=1であり,60\leqq A\leqq120のもとで,X_1=30,X_2=\frac{1}{2}\cdot60+\frac{1}{1}\cdot\left(A-\frac{1}{2}\cdot240+\frac{1}{2}\cdot120\right)=-30+Aより,X_2=4X_1となるAは存在しない(A=150となり,60\leqq A\leqq120に抵触).
    ケース3が適用されるなら,k=1であり,120\leqq A\leqq180のもとで,X_1=30,X_2=180-\frac{1}{2}\cdot60-\frac{1}{1}\cdot\left(\frac{1}{2}\cdot240+\frac{1}{2}\cdot120-A\right)=-30+Aより,X_2=4X_1となるAA=150
    ケース4が適用されるなら,180\leqq Aのもとで,X_1=\frac{1}{2}A-60,X_2=180-\frac{1}{2}\left(240-A\right)=60+\frac{1}{2}Aより,X_2=4X_1となるAA=200
    以上より,A=150,200……(答)

    (2)
    ケース1が適用されるなら,A\leqq\frac{3}{2}\cdot60\Leftrightarrow A\leqq90が必要だが,A=100A=220もこの範囲にない.
    ケース2が適用されるなら,

    が必要となる.A=10090\leqq A\leqq120の範囲内であるから,k=1とした式が成り立ち,
    X_1=30,X_2=12⋅60+12100-12⋅330+1230+120=35,X3=35となる.……(答)
    ケース3が適用されるなら,k=1に対して\frac{1}{2}\cdot330+\frac{1}{2}\cdot90\leqq A\leqq\frac{1}{2}\cdot330+\frac{1}{2}\left(30+120\right)\Leftrightarrow210\leqq A\leqq240が必要となる.A=220210\leqq A\leqq240の範囲内であるから,k=1とした式が成り立ち,
    X_1=30,X_2=90-\frac{1}{2}\cdot60-\frac{1}{2}\left\{\frac{1}{2}\cdot330+\frac{1}{2}\left(30+120\right)-220\right\}=50,X_3=180-\frac{1}{2}\cdot60-\frac{1}{2}\left\{\frac{1}{2}\cdot330+\frac{1}{2}\left(30+120\right)-220\right\}=140……(答)

2016年慶応義塾大学環境情報|過去問徹底研究 大問5

2019.09.23

方針の立て方 ガウス記号()の問題はとにかく書き出してみること.書き出していく中で規則性をつかむことができる.(1)の場合にはが平方数となる前後での値が1増えることが分かる.そのため,が平方数となる箇所ごとに数列を区切って,群数列としてみると良い(特にを求めるときに,分母が同じものに着目することが重

  • …続きを読む
  • 方針の立て方
    ガウス記号(\left[\qquad\right])の問題はとにかく書き出してみること.書き出していく中で規則性をつかむことができる.(1)の場合にはnが平方数となる前後で\left[\sqrt n\right]の値が1増えることが分かる.そのため,nが平方数となる箇所ごとに数列を区切って,群数列としてみると良い(特にS_{99}を求めるときに,分母が同じものに着目することが重要だと気付くだろう).同様に,(2)の場合にはnが立方数となる箇所ごとに数列を区切る.

    解答例
    (70)(71)……27
    (72)(73)……80
    (74)(75)(76)……714
    (77)(78)……46
    (79)(80)……20
    (81)(82)(83)(84)……2178

    解説
    (1)
    a_nが整数となるもの((70)と(71)について)
    分母の\left[\sqrt n\right]の値で場合分けする.
    \left[\sqrt n\right]=1となるのは,1\leqq n\leqq3であり,a_nが整数となるのは,n=1,2,3で3個.
    \left[\sqrt n\right]=2となるのは,4\leqq n\leqq8であり,a_nが整数となるのは,n=4,6,8で3個.
    \left[\sqrt n\right]=3となるのは,9\leqq n\leqq15であり,a_nが整数となるのは,n=9,12,15で3個.
    \left[\sqrt n\right]=4となるのは,16\leqq n\leqq24であり,a_nが整数となるのは,n=16,20,24で3個.
    \left[\sqrt n\right]=5となるのは,25\leqq n\leqq35であり,a_nが整数となるのは,n=25,30,35で3個.
    \left[\sqrt n\right]=6となるのは,36\leqq n\leqq48であり,a_nが整数となるのは,n=36,42,48で3個.
    \left[\sqrt n\right]=7となるのは,49\leqq n\leqq63であり,a_nが整数となるのは,n=49,56,63で3個.
    \left[\sqrt n\right]=8となるのは,64\leqq n\leqq80であり,a_nが整数となるのは,n=64,72,80で3個.
    \left[\sqrt n\right]=9となるのは,81\leqq n\leqq99であり,a_nが整数となるのは,n=81,90,99で3個.
    以上より,求める個数は,3\times9=27個……(答)

    ○最初にa_n=10となるn((72)と(73)について)
    分母の\left[\sqrt n\right]の値で場合分けする.
    \left[\sqrt n\right]=1となる項の中で最大の項は,n=3のときで,a_3=3
    \left[\sqrt n\right]=2となる項の中で最大の項は,n=8のときで,a_8=4
    \left[\sqrt n\right]=3となる項の中で最大の項は,n=15のときで,a_{15}=5
    \left[\sqrt n\right]=4となる項の中で最大の項は,n=24のときで,a_{24}=6
    \left[\sqrt n\right]=5となる項の中で最大の項は,n=35のときで,a_{35}=7
    \left[\sqrt n\right]=6となる項の中で最大の項は,n=48のときで,a_{48}=8
    \left[\sqrt n\right]=7となる項の中で最大の項は,n=63のときで,a_{63}=9
    \left[\sqrt n\right]=8となる項の中で最大の項は,n=80のときで,a_{80}=10
    よって,最初にa_n=10となるnn=80……(答)

    S_{99}((74)~(76)について)
    分母の\left[\sqrt n\right]の値が同じ項をまとめて考える(群数列の考え方).
    S_{99}=\sum_{i=1}^{99}a_i=\frac{1+2+3}{1}+\frac{4+5+\cdots\cdots+8}{2}+\frac{9+10+\cdots\cdots+15}{3}+\cdots\cdots+\frac{81+82+\cdots\cdots+99}{9}=\sum_{n=1}^{9}\frac{\sum_{m=n^2}^{\left(n+1\right)^2-1}m}{n}=\sum_{n=1}^{9}\frac{\frac{1}{2}\left(2n+1\right)\left\{n^2+\left(n+1\right)^2-1\right\}}{n}=\sum_{n=1}^{9}\left(2n^2+3n+1\right)=714……(答)

    (2)
    b_nが整数となるもの((77)と(78)について)
    分母の\left[\sqrt[3]{n}\right]の値で場合分けする.
    \left[\sqrt[3]{n}\right]=1となるのは,1\leqq n\leqq7であり,a_nが整数となるのは,n=1,2,3,\cdots\cdots,7で7個.
    \left[\sqrt[3]{n}\right]=2となるのは,8\leqq n\leqq26であり,a_nが整数となるのは,n=8,10,12,\cdots\cdots,26で10個.
    \left[\sqrt[3]{n}\right]=3となるのは,27\leqq n\leqq63であり,a_nが整数となるのは,n=27,30,33,\cdots\cdots,63で13個.
    \left[\sqrt[3]{n}\right]=4となるのは,64\leqq n\leqq124であり,a_nが整数となるのは,n=64,68,72,\cdots\cdots,124で16個.
    以上より,求める個数は,7+10+13+16=46個……(答)

    ○最初にb_n=10となるn((79)と(80)について)
    分母の\left[\sqrt[3]{n}\right]の値で場合分けする.
    \left[\sqrt[3]{n}\right]=1となる項の中で最大の項は,n=7のときで,b_7=7
    \left[\sqrt[3]{n}\right]=2となる項の中で最大の項は,n=26のときで,b_{26}=13
    よって求めるn\left[\sqrt[3]{n}\right]=2となる項の中にある.分母が2のため,分子が20になる項が該当する.そしてその項はb_{20}である.
    よって,最初にb_n=10となるnn=20……(答)

    T_{124}((81)~(84)について)
    分母の\left[\sqrt[3]{n}\right]の値が同じ項をまとめて考える(群数列の考え方).
    T_{124}=\sum_{i=1}^{124}b_i=\frac{1+2+\cdots\cdots+7}{1}+\frac{8+9+\cdots\cdots+26}{2}+\frac{27+28+\cdots\cdots+63}{3}+\frac{64+65+\cdots\cdots+124}{4}=\sum_{n=1}^{4}\frac{\sum_{m=n^3}^{\left(n+1\right)^3-1}m}{n}=\sum_{n=1}^{4}\frac{\frac{1}{2}\left(3n^2+3n+1\right)\left\{n^3+\left(n+1\right)^3-1\right\}}{n}=\sum_{n=1}^{4}{\frac{1}{2}\left(3n^2+3n+1\right)\left(2n^2+3n+3\right)}=\frac{1}{2}\cdot7\cdot8+\frac{1}{2}\cdot19\cdot17+\frac{1}{2}\cdot37\cdot30+\frac{1}{2}\cdot61\cdot47=2178……(答)

2016年慶應義塾大学環境情報|過去問徹底研究 大問4

2019.09.23

方針の立て方 (1) 頻出問題であるため,原理とともに解法をおさえておきたい.答えとなる点は直線上にあるわけだが,直線は線分の垂直二等分線であるから,直線上の点と点,点との距離は等しくなる.よって,点で考察するのと,点で考察するのは等価となる.これが,この問題(解法)の原理である. (2) 前問で説

  • …続きを読む
  • 方針の立て方
    (1)
    頻出問題であるため,原理とともに解法をおさえておきたい.答えとなる点\mathrm{P}は直線l上にあるわけだが,直線lは線分\mathrm{AA}^\primeの垂直二等分線であるから,直線l上の点と点\mathrm{A},点\mathrm{A}^\primeとの距離は等しくなる.よって,点\mathrm{A}で考察するのと,点\mathrm{A}^\primeで考察するのは等価となる.これが,この問題(解法)の原理である.
    (2)
    前問で説明した原理を応用すればよい.

    解答例
    (50)(51)(52)(53)……\frac{-11}{6}
    (54)(55)(56)(57)……\frac{017}{6}
    (58)(59)(60)……\frac{-5}{3}
    (61)(62)(63)……\frac{08}{3}
    (64)(65)(66)……\frac{21}{5}
    (67)(68)(69)……\frac{06}{5}

    解説
    (1)

    上図のように,直線lに対して点\mathrm{A}と対称な点を\mathrm{A}^\primeとする.
    直線\mathrm{AA}^\prime(図の破線)の式はy=x+6であるから,\mathrm{A}^\primeの座標は,\left(-3,3\right)と分かる.
    よって,直線\mathrm{A}^\prime B(図の鎖線)の式はy=-\frac{1}{7}x+\frac{18}{7}と分かる.直線\mathrm{A}^\prime \mathrm{B}と直線lの交点が点\mathrm{P}であり,その座標は,
    \left(\frac{-11}{6},\frac{17}{6}\right)……(答)

    (2)

    直線mに対して点\mathrm{B}と対称な点を\mathrm{B}^\primeとする.前問と同様に点\mathrm{B}^\primeの座標を求めると,\left(5,1\right)となる.
    よって,直線\mathrm{A}^\prime \mathrm{B}^\prime(図の鎖線)の式はy=-\frac{1}{4}x+\frac{9}{4}と分かる.直線\mathrm{A}^\prime \mathrm{B}^\primeと直線l,mの交点が点\mathrm{P},\mathrm{Q}であり,その座標は,\mathrm{P}\left(-\frac{5}{3},\frac{8}{3}\right),\mathrm{Q}\left(\frac{21}{5},\frac{6}{5}\right)……(答)


  • 偏差値30からの早慶圧勝の個別指導塾 HIRO ACADEMIA

  • 早稲田校舎 : 〒162-0045
    東京都新宿区馬場下町9-7 ハイライフホーム早稲田駅前ビル4階
    TEL: 03-6884-7991
    営業時間: 月〜土 9:00-21:30 
  • Facebook Twitter
    Page Top

Copyright © BETELGEUSE corporation All Rights Reserved.